Đến nội dung

Hình ảnh

$3(a^4+b^4+c^4)+a^2+b^2+c^2+6\geq 6(a^3+b^3+c^3)$

- - - - -

  • Please log in to reply
Chủ đề này có 5 trả lời

#1
longatk08

longatk08

    Sĩ quan

  • Thành viên
  • 350 Bài viết

Bài toán: Cho $a,b,c$ là các số thực không âm thỏa $a+b+c=3$.Chứng minh rằng:

 

$3(a^4+b^4+c^4)+a^2+b^2+c^2+6\geq 6(a^3+b^3+c^3)$

 

Spoiler


Bài viết đã được chỉnh sửa nội dung bởi longatk08: 06-06-2015 - 22:31


#2
dogsteven

dogsteven

    Đại úy

  • Thành viên
  • 1567 Bài viết

Ta dùng phương pháp tiếp tuyến và bất đẳng thức Schur suy rộng:

Bất đẳng thức đã cho tương đương với:

$\sum (a-1)^2(3a^2-2)\geqslant 0\Leftrightarrow \sum (2a-b-c)^2(3a^2-2)\geqslant 0\Leftrightarrow \sum (4a^2+b^2+c^2-4)(a-b)(a-c)\geqslant 0$

Áp dụng bất đẳng thức Cauchy-Schwarz: $4a^2+b^2+c^2-4\geqslant 4a^2+\dfrac{(3-a)^2}{2}-4\geqslant 0$

Giả sử $a\geqslant b\geqslant c\geqslant 0$ thì $4a^2+b^2+c^2-4\geqslant 4b^2+c^2+a^2-4$ nên bất đẳng thức đúng.


Bài viết đã được chỉnh sửa nội dung bởi dogsteven: 07-06-2015 - 06:50

Quyết tâm off dài dài cày hình, số, tổ, rời rạc.


#3
binhnhaukhong

binhnhaukhong

    Sĩ quan

  • Thành viên
  • 343 Bài viết

Còn 1 cách nữa khá nhọ là đổi biến để dùng BĐT Schur  :biggrin:


Quy Ẩn Giang Hồ. 

So goodbye!

 

:off:  :off:  :off:  :off:  :off:  :off: 


#4
khanghaxuan

khanghaxuan

    Trung úy

  • Thành viên
  • 969 Bài viết

Dùng đồng bậc cũng được :)) 


Điều tôi muốn biết trước tiên không phải là bạn đã thất bại ra sao mà là bạn đã chấp nhận nó như thế nào .

- A.Lincoln -

#5
khanghaxuan

khanghaxuan

    Trung úy

  • Thành viên
  • 969 Bài viết

Cách dùng đồng bậc :  

Sau khi đồng bậc 4 cho bđt , khai triển rồi rút gọn thì cuối cùng ta cần chứng minh : 

$32\sum a^{4}+18\sum a^{2}b^{2}+30abc\sum a\geq 40\sum ab(a^{2}+b^{2})$

Đến dùng biến đổi SOS ta được : 

$\sum (a-b)^{2}(16a^{2}+16b^{2}-8ab-15c^{2})\geq 0 (*)$

Dễ dàng nhận thấy $(*)$ đúng :))


Điều tôi muốn biết trước tiên không phải là bạn đã thất bại ra sao mà là bạn đã chấp nhận nó như thế nào .

- A.Lincoln -

#6
Nguyenhuyen_AG

Nguyenhuyen_AG

    Trung úy

  • Thành viên nổi bật 2016
  • 945 Bài viết

Bài toán: Cho $a,b,c$ là các số thực không âm thỏa $a+b+c=3$.Chứng minh rằng:

 

$3(a^4+b^4+c^4)+a^2+b^2+c^2+6\geq 6(a^3+b^3+c^3). \quad (2.2.1)$

 

Spoiler

 

Đặt $a=x+1,\,b=y+1,\,c=z+1,$ khi đó \[x+y+z= (a-1) + (b-1) + (c-1) = 0.\] Với phép đặt này thì \[\begin{aligned} a^2+b^2+c^2&=(x+1)^2+(y+1)^2+(z+1)^2\\&=x^2+y^2+z^2+2(x+y+z)+3\\&=x^2+y^2+z^2+3, \end{aligned}\] tương tự \[\begin{aligned} a^3+b^3+c^3& = x^3+y^3+z^3+3(x^2+y^2+z^2)+3, \\ a^4+b^4+c^4& =x^4+y^4+z^4+4(x^3+y^3+z^3)+6(x^2+y^2+z^2)+3. \end{aligned}\] Bất đẳng thức (2.2.1) trở thành \[3(x^4+y^4+z^4)+6(x^3+y^3+z^3)+x^2+y^2+z^2 \ge 0. \quad (2.2.2)\] Giả sử $xy \ge 0,$ rồi thay $z=-x-y$ vào (2.2.2), ta được \[3[x^4+y^4+(x+y)^4]+6[x^3+y^3-(x+y)^3]+x^2+y^2+(x+y)^2 \ge 0,\] \[3(x^4+2x^3y+3x^2y^2+2xy^3+y^4)+x^2+xy+y^2\ge 9xy(x+y)\] \[3(x^2+xy+y^2)^2+x^2+xy+y^2\ge 9xy(x+y).\] Áp dụng bất đẳng thức AM-GM, ta có \[x^2+xy+y^2 \ge \frac{3}{4}(x+y)^2 \ge 3xy \ge 0, \quad (2.2.3)\] suy ra \[3(x^2+xy+y^2)^2+x^2+xy+y^2\ge 27x^2y^2+\frac{3}{4}(x+y)^2.\] Ta chứng minh \[9x^2y^2+\frac{(x+y)^2}{4} \ge 3xy(x+y).\] Cũng theo bất đẳng thức AM-GM, thì \[9x^2y^2+\frac{(x+y)^2}{4} \ge 2\sqrt{9x^2y^2\cdot\frac{(x+y)^2}{4}}=3xy\left | x+y \right |\ge 3xy(x+y). \quad (2.2.4)\] Đẳng thức xảy ra khi (2.2.3) và (2.2.4) trở thành đẳng thức, tức $x,\,y$ là nghiệm của hệ \[\left\{ \begin{aligned} & x=y \\ & 27x^2y^2 = \frac{3}{4}(x+y)^2 \end{aligned}\right.\] Giải hệ này ta được $x=y=0$ hoặc $x=y=\frac{1}{3},$ suy ra $a=b=c=1,$ hoặc $a=b=\frac{4}{3},\,c=\frac{1}{3}$ cùng các hoán vị. Bài toán được chứng minh.


Bài viết đã được chỉnh sửa nội dung bởi Nguyenhuyen_AG: 16-06-2015 - 22:05

Nguyen Van Huyen
Ho Chi Minh City University Of Transport




1 người đang xem chủ đề

0 thành viên, 1 khách, 0 thành viên ẩn danh